2
$\begingroup$

I have tried to solve this exercise:

Let $H$ be a Hilbert space and let $T = T^* \in B_{\infty}(H)$ a compact operator. Given $\psi_0 \in H$ consider the equations: \begin{align*} (1)\quad &T\psi = \psi,\\ (2)\quad &T\psi' = \psi' + \psi_0. \end{align*}

Show that if the only solution of (1) is $\psi = 0$, then equation (2) has a unique solution.

When I was done solving it, I found out that I didn't know whether or not the hypothesis of T being compact was useful. Can you help me find out if it is needed somewhere in my proof?

Here is the proof:

$\exists !\psi \in H$ | $T\psi = \psi = 0 \implies$ $\psi \in$ ker($T$) $= \{0\} \implies (T-I)$ is injective.

ker$(T-I)^{\bot} = $ Ran$(T^{*}- I) = $ Ran$(T - I) = $ $\{0\}^{\bot} = H \implies T$ is surjective $\implies T$ is bijective.

Then $\forall \psi_0 \in H $, $\exists! \psi' \in H $ so that $(T - I)\psi' = \psi' + \psi_0 \implies$ (2) has a unique solution.

Thanks in advance for your help!!!

$\endgroup$

1 Answer 1

3
$\begingroup$

For a bounded operator you have $(ker(T- I))^\perp = \overline{Ran(T^*- I)}$. But , since $T=T^*$ is compact we have that $Ran(T^*- I)$ is closed, so $(ker(T- I))^\perp = Ran(T^*- I)$.

A faster way to solve this is: $(1)$ says that $1$ is not an eigenvalue of $T$. Since $T$ is compact and self-adjoint we conclude that $1$ is not in the spectrum of $T$, so $T-I$ is invertible.

Added. Claim: If $T$ is compact then $Ran(T-I)$ is closed.

Suppose $T(x_n)-x_n\rightarrow_n y.$ WLOG we can assume that $x_n \in Ker(T -I)^\perp$. The sequence $x_n$ is bounded. Indeed otherwise taking a subsequence we can assume $\lim_n \|x_n\|=\infty$. Then

$T(\frac{x_n}{\|x_n\|})-\frac{x_n}{\|x_n\|}\rightarrow_n 0.$

Since $T$ is compact WLOG we can assume that $\lim_n T(x_n/\|x_n\|) = z\in (ker(T- I))^\perp$. But this implies

$$\lim_n \frac{x_n}{\|x_n\|} =z\in (ker(T- I))^\perp$$

so $\|z\|=1$ and $T(z)-z=0$, that contradicts $z\in (ker(T- I))^\perp$ and $z\neq 0$. So $x_n$ is bounded.

Taking a subsequence we can assume that $\lim_n T(x_n) = w\in H$. This implies $\lim_n x_n = u := w-y.$ and $Tu -u = y$. So $Ran (T -I)$ is closed.

$\endgroup$
2
  • $\begingroup$ Can you specify why I can say that the $Ran(T - I)$ is closed? $\endgroup$ Commented Nov 22 at 12:47
  • $\begingroup$ I added a proof. $\endgroup$ Commented Nov 22 at 13:17

You must log in to answer this question.

Start asking to get answers

Find the answer to your question by asking.

Ask question

Explore related questions

See similar questions with these tags.